Fundamentals of Nursing Science

¡Supera tus tareas y exámenes ahora con Quizwiz!

Before endotracheal suctioning of a patient, the nurse should perform which intervention? 1. Have the patient take several deep breaths. 2. Instruct the patient to cough before suctioning. 3. Administer 100% oxygen to the patient. 4. Change the suctioning equipment to ensure sterility.

3. Administer 100% oxygen to the patient. Rationale: Before suctioning, regardless of the means, oxygen should be administered, because the suctioning procedure depletes oxygen from the respiratory tract, causing a potential drop in oxygen saturation levels. In a patient with an endotracheal tube, manually bagging with 100% oxygen will hyperoxygenate the lungs. The patient who has an endotracheal tube may not be able to follow commands to take deep breaths, cough, or have the strength to do either, which is why manual bagging is preferred. A new sterile suction catheter should be used each time the patient is suctioned, but the suction tubing and equipment need not be changed.

Which assessment finding for a patient who is receiving therapy for vitamin B12 deficiency should indicate to the nurse that the therapy is having the desired effect? 1. Normal serum electrolyte levels 2. Healthy skin integrity 3. Resolution of peripheral edema 4. Improved hemoglobin and hematocrit levels

4. Improved hemoglobin and hematocrit levels Rationale: Vitamin B12 is essential for appropriate maturation of red blood cells; therefore relieving the deficiency is expected to improve hemoglobin and hematocrit (H&H) levels and decrease hypoxia-related problems. This disorder is known as pernicious anemia. Normal serum electrolytes, healthy skin integrity, and resolution of peripheral edema if present would be secondary to improved hemoglobin and hematocrit levels.

Carbon monoxide poisoning

Blood level 1-10% - normal level Blood level 11-20% - headache, flushing, decreased vision, decreased cerebral function, slight breathlessness Blood level 21-40% - headache, nausea/vomiting, drowsiness, vertigo, confusion, pale skin, hypotension, tachycardia Blood level 41-60% - coma, seizures Blood level 61-80% - death

Nursing Process

1. Assessing - collecting data 2. Diagnosing - figuring out what is the problem 3. Planning - how to best manage the problem 4. Implementing- putting the plan into action 5. Evaluating- did the plan work?

A nurse understands that the primary purpose for a patient to undergo reconstructive surgery is to: 1. Restore function and/or appearance. 2. Replace an organ or tissue. 3. Relieve or reduce symptoms. 4. Remove or excise an organ or tissue.

1. Restore function and/or appearance. Rationale: The main function of reconstructive surgery is to restore function and/or appearance. This type of surgery includes plastic surgery, a term that is interchangeable with reconstructive surgery. In reconstructive surgery, repairs are made and malformations corrected that are congenital, a result of disease processes, or from traumatic injury. Replacement of a tissue or organ (2) is known as transplant; to relieve or reduce symptoms (3) is known as palliative; and to remove or excise an organ or tissue is known as resection.

A patient with a history of severe diarrhea for the past 3 days is admitted for dehydration. Which intravenous solution would the nurse anticipate the healthcare provider to order initially for this patient? 1. 3% sodium chloride 2. 0.9% sodium chloride 3. 5% dextrose and 0.9% sodium chloride 4. 5% dextrose and lactated Ringer's

2. 0.9% sodium chloride Rationale: An IV solution of 0.9% sodium chloride is the most appropriate initial IV fluid for this patient, because it is an isotonic solution that will act as a volume expander to quickly replace volume losses and promote physiological stabilization. 3% sodium chloride is a high concentration (hypertonic) electrolyte solution; it would only be used in a patient with hyponatremia and must be closely monitored during infusion. 5% dextrose and 0.9% sodium chloride and 5% dextrose and lactated Ringer's may be appropriate fluids to infuse after 0.9% sodium chloride.

A nurse is performing preoperative teaching for a patient and gives instructions regarding use of an incentive spirometer. Which procedure for using this device is correct? 1. Inhale completely and exhale in short, rapid breaths. 2. Inhale deeply through the spirometer, hold it as long as possible, and slowly exhale. 3. Exhale completely; take a slow, deep breath; hold it as long as possible, and slowly exhale. 4. Exhale halfway, then inhale a rapid, small breath; repeat several times.

2. Inhale deeply through the spirometer, hold it as long as possible, and slowly exhale. Rationale: The correct procedure to maximize use of an incentive spirometer is to exhale completely, then take a slow, deep breath through the spirometer, and hold it as long as possible. This procedure will maximize inspiratory function by expanding the lungs. The patient should practice using the incentive spirometer before surgery. Answer options 1 and 4 are completely inaccurate procedures for using an incentive spirometer. Option 3 is partially correct but does not state to use the incentive spirometer. When teaching patients, it is important to provide exact step-by-step instructions, thus not leaving out any critical points.

Intramuscular injection

-Gluteus, thigh, and deltoid -23 G needle -90 degree angle -Up to 3 mL injected (in deltoid, no more than 1 mL)

Intradermal injection

-Inner forearm, chest, and back -27-30 G needle -10-15 degree angle -0.1 - 0.2 mL injected

Visually impaired patient

-Knock on door. -Introduce self. -Use clear and simple sentences. -Stay within pt's field of vision. -Orient pt to room + call bell. -Explain unusual noises. -Inform of location of food on meal tray using numbers on a clock. -When ambulating the pt, walk slightly in front of pt while pt holds your arm.

Fall prevention

-Orient to room. -Teach how to use call bell and when to use call bell. -Keep bed in lowest position. -Raise 3 of 4 side rails. -Lock wheels on all equipment. -Keep floor dry and free of cords. -Wear slip resistant footwear. -Stay with pt during shower.

Subcutaneous injection

-Outer upper arm, anterior thigh, and abdomen -25-28 G needle -90 degree angle -0.5-1 mL injected

Deep tendon reflexes

0 = no response 1+ = diminished 2+ = active/expected response 3+ = slightly hyperactive 4+ = brisk/hyperactive with intermittent clonus

Pulse grades

1+ - Weak and barely palpable 2+ - Normal and easily palpable 3+ - Full pulse and increased 4+ - Strong and bounding

A nurse should employ which technique to maintain surgical asepsis? 1. Change the sterile field after sterile water is spilled on it. 2. Put on sterile gloves, then open a container of sterile saline. 3. Place a sterile dressing no more than 1/2 inch from the edge of the sterile field. 4. Clean the surgical area with a circular motion, moving from the outer edge toward the center.

1. Change the sterile field after sterile water is spilled on it. Rationale: A sterile field is considered contaminated when it becomes wet. Moisture can act as a wick and allow microorganisms to contaminate the field. The outsides of containers and packages are not considered sterile, and sterile gloves are considered contaminated when touching either of these items (2). Items on the sterile field should be no more than one inch from the outer border or edge of the sterile field (3); any less is not considered sterile. Surgical areas or wounds should be cleaned from the inside edges to the outside edges (4) to prevent recontamination.

A physician has inserted a central venous catheter (CVC) in a patient. What would the nurse anticipate being required for the patient before using the catheter? 1. Chest x-ray. 2. Flushing the line with heparin flush. 3. Withdrawing blood to ensure patency. 4. Chest fluoroscopy.

1. Chest x-ray. Rationale: The insertion of a central venous catheter (CVC) into the subclavian vein can result in a pneumothorax, which would be seen on a chest x-ray. Indications of a pneumothorax before the chest x-ray would include shortness of breath and anxiety. If the chest x-ray is negative for pneumothorax, the CVC line may be used. The central line should not be flushed until placement is verified. Blood withdrawal is utilized once placement is verified but it is not used to verify initial placement. Fluoroscopy may be used during placement in certain settings but not for placement verification.

A nurse is instructing a community group regarding risk factors for coronary artery disease. Which risk factor should the nurse emphasize as NOT modifiable? 1. Heredity. 2. Hypertension. 3. Cigarette smoking. 4. Diabetes mellitus.

1. Heredity Rationale: Heredity refers to genetic makeup and cannot be changed. Cigarette smoking is a lifestyle habit that involves behavior modification. Hypertension and diabetes mellitus are risk factors of coronary artery disease that can be controlled with diet, medication, and exercise.

A nurse is caring for a patient with pneumonia. Which of the following interventions is the highest priority? 1. Increase fluid intake. 2. Employ breathing exercises and controlled coughing. 3. Ambulate as much as possible. 4. Maintain an NPO status.

2. Employ breathing exercises and controlled coughing. Rationale: For most patients, the most effective means of preventing fluid consolidation in the lungs with a diagnosis of pneumonia is to keep active by deep breathing and controlled coughing exercises. Increased fluid intake and ambulation are important aspects of care if not contraindicated, but they are secondary to deep breathing and coughing. Keeping the patient NPO is not necessary; unless contraindicated, the patient with pneumonia is usually offered their regular diet as tolerated.

The nurse teaching a health awareness class identifies which situation as being the highest risk factor for the development of a deep vein thrombosis (DVT)? 1. Pregnancy 2. Inactivity 3. Aerobic exercise 4. Tight clothing

2. Inactivity Rationale: A DVT, or thrombus, may form as a result of venous stasis. It may lodge in a vein and can cause venous occlusion. Inactivity is a major cause of venous stasis leading to DVT. Pregnancy and tight clothing are also risk factors for DVT secondary to inactivity. Aerobic exercise is not a risk factor for DVT.

Which landmark is correct for a nurse to use when auscultating the mitral valve? 1. Left 5th intercostal space, midaxillary line. 2. Left 5th intercostal space, midclavicular line. 3. Left 2nd intercostal space, sternal border. 4. Left 5th intercostal space, sternal border.

2. Left 5th intercostal space, midclavicular line. Rationale: The correct landmark for auscultating the mitral valve (apical pulse) is found at the left 5th intercostal space in the midclavicular line. Auscultation at the 5th intercostal space (ICS), midaxillary line would yield breath sounds of the lateral lung field. Auscultation at the left 2nd intercostal space sternal border is best to hear the pulmonic valve, and at the left 5th ICS sternal border for the tricuspid valve.

Upon auscultation, the nurse hears abnormal breath sounds that are high pitched, creaking, and accentuated on expiration. Which term best describes these data? 1. Rhonchi 2. Wheezes 3. Pleural friction rub 4. Bronchovesicular

2. Wheezes Rationale: Wheezes are one of the most common breath sounds assessed and auscultated in patients with asthma and COPD. Wheezes are produced as air flows through narrowed passageways. Rhonchi are coarse rattling sounds similar to snoring and are usually caused by secretions in the bronchial airways. A pleural friction rub is an abrasive sound made by two acutely inflamed serous surfaces rubbing together during the respiratory cycle. Bronchovesicular sounds are intermediate between bronchial (upper) and vesicular (lower) breath sounds; they are normal when heard between the 1st and 2nd intercostal spaces anteriorly and posteriorly between scapulae.

Which food should the patient with a colostomy be advised to avoid most? 1. Milk 2. Cheese 3. Coffee 4. Cabbage

4. Cabbage Rationale: Cabbage is a gas-producing food that can cause a patient with a colostomy problems with odor control and ballooning of the ostomy bag, which may break the device seal and allow leakage. Milk, cheese, and coffee should not cause excessive gas problems in moderation. The patient with a new colostomy should slowly introduce new foods into the diet to test toleration.

A nurse is assessing a patient who had a lung resection for a malignancy. The patient had just returned to the nursing unit with a closed chest tube drainage system connected to suction. Which finding would require additional evaluation in the initial postoperative period? 1. A column of water 20 cm high in the suction control chamber 2. 75 mL of bright red blood in the drainage collection chamber 3. An intact occlusive dressing at the insertion site 4. Constant bubbling in the water seal chamber

4. Constant bubbling in the water seal chamber Rationale: Constant bubbling in the water seal chamber is indicative of an air leak. The nurse should assess the entire length of

Which action by a home care nurse would be considered an act of euthanasia? 1. Implementing a "do not resuscitate" order in the home health setting. 2. Abiding by the decision of a living will signed by the patient's family. 3. Encouraging a patient to consult an attorney to document and assign a power of attorney. 4. Knowing that a dying patient is overmedicating and not acting on this information.

4. Knowing that a dying patient is overmedicating and not acting on this information. Rationale: In this situation being aware that a patient is overmedicating and taking no action can be considered an act of euthanasia on the part of the home care nurse. Implementing a "do not resuscitate" order, abiding by the decision of a living will signed by the patient's family, and encouraging the patient to consult an attorney are all appropriate actions for a home care nurse.

ABC's

Airway - Is it clear? Breathing - If this isn't possible, O2 won't reach the lungs and be circulated around the body. Circulation - Without circulation, hypoxia and cardiac arrest will ensue

Ambulating with cane

Going up stairs: Strong leg, cane, weak leg Going down stairs: Cane, weak leg, strong leg -Cane always moves before the weak leg!

Fire safety

RACE 1. R - Rescue puts in danger. 2. A - Activate alarm. 3. C - Contain fire (close window/doors). 4. E - Extinguish fire if small.

Peripherally inserted central catheter (PICC)

Usage: long term IV abx, chemotherapy treatment, nutritional support/TPN Priority: 1. Monitor for infection, embolism, and occlusion 2. Sterile dressing changes 3. Have pt hold breath during tube change 4. Have pt turn head away from PICC during PICC line use 5. If air embolism suspected - Trendelenburg + L side lying

Contact precautions

Used for direct patient or environmental contact with blood or body fluids from an infected patient. This includes colonization of infection with multidrug-resistant organisms (MDRO) such as MRSA, stool infected with C.diff, draining wounds where secretions are not contained, or scabies

A nurse caring for a patient with a nosocomial infection caused by methicillin-resistant Staphylococcus aureus (MRSA). The patient has a new colostomy for which the nurse is preparing to provide care. Which protective items should the nurse use to perform the client's care? Select all that apply. 1. Gloves 2. Gown 3. Mask 4. Goggles 5. Shoe covers 6. Hair bonnet

1, 2, 4: Gloves, gown, and goggles Rationale: Standard personal protective equipment (PPE), which should be used for performing colostomy care in a client positive for MRSA, includes gloves, gown, and goggles. A combination mask/eye shield may be used when caring for this client; however, a mask is not necessary. Shoe covers and hair bonnet are not required for this situation.

A nurse assesses the vital signs of a 50-year-old female patient and documents the results. Which of the following are considered within normal range for this patient? 1. Oral temperature 98.2 degrees F 2. Apical pulse 88 beats per minute and regular 3. Respiratory rate of 30 per minute 4. Blood pressure 116/78 mm Hg while in a sitting position 5. Oxygen saturation of 92%

1, 2, and 4 Rationale: The patient's temperature, pulse, and BP are within normal ranges for a 50-year-old female. The patient's respirations are mildly elevated (3) and the oxygen saturation level is below normal (5). A normal respiratory rate for a female patient in this age-group would be 12 to 20 per minute, and oxygen saturation level should be >95%.

A nurse discusses the procedure for protective environment isolation with the husband of a patient who is receiving chemotherapy and has been hospitalized for neutropenia. Which statement made by the husband indicates the teaching was effective? 1. "Protective environment isolation helps prevent the spread of infection to my wife from outside sources." 2. "Protective environment isolation helps prevent the spread of infection from my wife to others." 3. "Protective environment isolation helps prevent the spread of infection from my wife by using special techniques to destroy infectious fluids and secretions." 4. "Protective environment isolation helps prevent the spread of infection to my wife by using special sterilization techniques for her linens and personal items before use."

1. "Protective environment isolation helps prevent the spread of infection to my wife from outside sources." Rationale: Protective environment isolation implies that the activities and actions of the nurse will protect the patient from infectious agents because the patient's own immune defense ability is compromised (neutropenia). Protective environment isolation is also referred to as reverse isolation. The other answer options (2,3,4) are incorrect concepts related to protective environment isolation.

The professional obligation of a nurse to assume responsibility for actions is referred to as: 1. Accountability 2. Individuality 3. Responsibility 4. Bioethics

1. Accountability Rationale: Nurses have an obligation to uphold the highest standards of practice, assume full responsibility for actions, and maintain quality in the knowledge base and skill of the profession. Individuality and responsibility are positive characteristics of the nurse but are not necessarily professional obligations. Bioethics is a field of study concerned with the ethics and philosophical implications of certain biological and medical procedures, treatments, and so on.

A patient is determined to be having an impending anaphylactic reaction secondary to a drug hypersensitivity. What should be the first action for the nurse to perform? 1. Administer O2. 2. Insert an IV catheter. 3. Take the vital signs. 4. Obtain an arterial blood gas analysis.

1. Administer O2. Rationale: Administering O2 should be the first action of the nurse for this patient. With anaphylaxis there is bronchial constriction and subsequent vascular collapse, therefore the airway is of primary concern. The vital signs should be checked after beginning the administration of O2. At this point, it would be appropriate to insert an IV catheter to administer emergency medications and possibly obtain an arterial blood gas analysis to determine oxygenation status.

A patient who experienced extensive burns is receiving IV fluids to replace fluid loss. The nurse should monitor for which initial sign of fluid overload? 1. Crackles in the lungs. 2. Decreased heart rate. 3. Increased blood pressure. 4. Decreased blood pressure.

1. Crackles, or rales, in the lungs are an early sign of pulmonary congestion and edema caused by fluid overload. Patients with fluid overload will usually demonstrate an increased heart rate and increased blood pressure. A decreased heart rate and decreased blood pressure in a patient with fluid overload would be a very late and fatal sign.

A nurse is teaching a community group about the basics of nutrition. A participant questions why fluoride is added to drinking water. The nurse should respond that it is a necessary element added to drinking water to promote: 1. Dental health. 2. Growth and development. 3. Improved hearing. 4. Night vision.

1. Dental health. Rationale: Fluoride, or fluorine, is an element necessary for good dental health that helps to harden tooth enamel and decrease dental caries. Natural food sources may not be adequate; therefore fluoride is added to drinking water to help meet the recommended daily requirements. Growth and development, hearing, and night vision are not direct influences of fluorine intake.

A patient with limited mobility is ready for discharge. Which instruction should the nurse emphasize with the patient to prevent urinary stasis and formation of renal calculi? 1. Increase oral fluid intake to 2-3 L per day. 2. Maintain bed rest after discharge. 3. Limit fluid intake to 1 L/day. 4. Void at least every hour.

1. Increase oral fluid intake to 2-3 L per day. Rationale: Increasing oral fluid intake to 2-3 L per day, if not contraindicated, will dilute urine and promote urine flow, thus preventing stasis and complications such as renal calculi. Bed rest and limited fluid intake may lead to urinary stasis and increase risk for the formation of renal calculi. Voiding at least every hour had no effect on urinary stasis and renal calculi.

During an admission assessment the nurse discovers a stage 1 pressure ulcer on a patient. Which of the following actions should the nurse implement immediately? 1. Turning and repositioning the patient every 2 hours. 2. Covering with an occlusive transparent dressing. 3. Cleaning with hydrogen peroxide and leaving it open to the air. 4. Providing the patient with a diet high in vitamin C, zinc, and protein.

1. Turning and repositioning the patient every 2 hours. Rationale: Turning and repositioning immobile patients at least every 2 hours is the best initial nursing action for preventing further skin breakdown. Other measures should also be taken to relieve pressure on the area to prevent progression and promote healing. Covering the area with an occlusive transparent dressing or cleansing the area with hydrogen peroxide are not recommended for this situation. Providing a diet high in vitamin C, zinc, and protein will also aid in tissue healing and help prevent further breakdown but it is not the priority action.

A new nurse in orientation is approached by a surveyor from the department of health and is asked, "What is the best means to prevent the spread of infection?" What is the best answer for the nurse to provide? 1. "Let me get my preceptor." 2. "Wash your hands before and after any patient care." 3. "Clean all instruments and work surfaces with an approved disinfectant." 4. "Ensure proper disposal of all items contaminated with blood or bodily fluids."

2. "Wash your hands before and after any patient care." Rationale: The best means to prevent the spread of infection is to break the chain of infection. This is most easily accomplished by the simple act of handwashing before and after all patient contact. The responses in answer options 1 and 3 may be correct, but they are not the best responses for this situation. It is not necessary that all items contaminated with blood or body fluids be disposed.

A nurse receives reports on the following patients. Which patient should the nurse assess first? 1. 25-year-old male patient with a hemoglobin of 15.9 2. 56-year-old female patient on warfarin (Coumadin) with an INR of 7.5. 3. 38-year-old female patient with a serum calcium level of 9.4. 4. 45-year-old male patient with a blood urea nitrogen (BUN) of 20 and a creatinine of 1.1.

2. 56-year-old female patient on Coumadin with an INR of 7.5 Rationale: The 56-year-old patient on warfarin (Coumadin) with an INR of 7.5 should be assessed first by the nurse, because this is an elevated result. Normal is considered between 2 and 3. This result is not therapeutic and the nurse should assess for bleeding and hemodynamic stability. The nurse should report the result to the physician and implement bleeding precautions. The other results are within normal ranges: hemoglobin for a 25-year-old male is 14-18 g/dl; serum calcium is 9.0-10.5 mg/dl; BUN is 5-20 mg/dl and creatinine is 0.7-1.5 mg/dl.

A nurse assesses a patient and determines that he has a closed soft tissue injury. Which term might the nurse use to describe the injury? 1. An abrasion. 2. A contusion. 3. A laceration. 4. An avulsion.

2. A contusion. Rationale: Closed wounds are considered contusions and hematomas because the skin is not broken. Abrasions, lacerations, and avulsions are considered open because there is a break in the skin integrity.

A hospitalized patient is scheduled to have a sigmoidoscopy. What should the nurse anticipate will be ordered prior to the procedure? 1. Keep the client NPO until after the procedure. 2. Administer a Fleet enema 1 hour before procedure. 3. Encourage increased intake of clear fluids. 4. Administer morphine a half hour before the procedure as a preop.

2. Administer a Fleet enema 1 hour before procedure Rationale: To facilitate visualization of the rectum and the sigmoid colon, the lower colon must be emptied immediately before the procedure. A Fleet or tap water enema should be used. The patient would be kept NPO for at least 8 hours before until after the procedure (1, 3). Morphine (4) is not typically used as a pre-op medication before a sigmoidoscopy.

When providing care for a client with a nasogastric (NG) tube, the nurse should take measures to prevent what serious complication? 1. Skin breakdown 2. Aspiration pneumonia 3. Retention ileus 4. Profuse diarrhea

2. Aspiration pneumonia Rationale: Of the choices provided, the potential complication of highest risk for a patient with an NG tube is aspiration pneumonia. Care should be taken to prevent dislodging of the tube or vomiting. Proper positioning of the patient with an NG tube would include supine or side-lying semi-Fowler's or higher. Skin breakdown in a patient with an NG tube may result from pressure of the tube against nasal structures. The tube should be periodically repositioned and taped to prevent this complication. A retention ileus is not related to an NG tube. A patient who develops profuse diarrhea with an NG tube requires further investigation. It may be totally unrelated or a result of an enteral feeding incompatibility.

Which nursing intervention is most appropriate for a patient in skeletal traction? 1. Add and remove weights as the patient desires. 2. Assess the pin sites at least every shift and as needed. 3. Ensure that the knots in the rope catch on the pulley. 4. Perform ROM to joints proximal and distal to the fracture at least once a day.

2. Assess the pin sites at least every shift and as needed. Rationale: Nursing care for a patient in skeletal traction may include assessing pin sites every shift and as needed. The needed weight for a patient in skeletal traction is prescribed by the physician and not as desired by the patient. The nurse also should ensure that the knots do not catch on the pulley and move freely. The performance of ROM is indicated for all joints except the ones proximal and distal to the fracture, since this area is immobilized by the skeletal traction to promote healing and further prevent injury and pain.

During an assessment the nurse pulls up on the patient's skin, releases it, and looks to see if the skin returns immediately to it's original position. What is this assessment technique? 1. Assessing pain tolerance 2. Checking skin turgor 3. Checking for ecchymosis formation 4. Measuring tissue mass

2. Assessing skin turgor Rationale: Skin turgor is assessed by gently pinching the skin and releasing it while observing the degree of elasticity. If the skin pinch remains elevated or is slow to return to its original position, this may be an indication of dehydration or deficient fluid volume. This assessment technique is not appropriate for assessing pain tolerance, checking for ecchymosis formation, or measuring tissue mass.

A nurse who promotes freedom of choice for clients in decision-making best supports which principle? 1. Justice 2. Autonomy 3. Beneficence 4. Paternalism

2. Autonomy Rationale: The principle of autonomy relates to the freedom of a person to form his or her own judgments and actions. The nurse promotes autonomy nonjudgmentally so as not to infringe on the decisions or actions of others. Justice means to be righteous, equitable, and to act or treat fairly. Beneficence relates to the state or act of doing good, being kind and charitable. It also includes promotion of well-being and abstaining from the injuring of others. Paternalism encompasses the practice of governing people in a fatherly manner, especially by providing for their needs without infringing on their rights or responsibilities.

A 58-year-old patient presents to the ER with a nosebleed. After applying pressure, what is the next nursing action? 1. Collect a medication history. 2. Check the BP 3. Instruct not to pick the nose. 4. Check the pulse.

2. Check the BP. Rationale: Nosebleeds can be indicative of high BP in an adult. Of the choices provided, the first action of the nurse should be to check the patient's BP. If elevated, the nurse can initiate measures to decrease the BP. The other options are appropriate, but not the highest priority. A medication history is critical to determine if the patient is on any anticoagulation therapy. After assessment and care, patient teaching might include instruction not to pick the nose. After the BP is measured, checking the pulse rate would be performed as part of the general vital signs assessment.

The nurse will be changing the soiled bed linens of a patient with a stage III pressure ulcer that is draining seropurulent material. What personal protective equipment (PPE) is most essential for the nurse to wear? 1. Mask 2. Clean gloves 3. Sterile gloves 4. Shoe covers

2. Clean gloves Rationale: Clean globes protect the hands and wrists from microorganisms in the linens. Clean gloves are the first line of defense in preventing the spread of infection. Mask, sterile gloves, and shoe covers are not required for this situation.

Upon returning from lunch, a nurse is approached in the elevator by a hospital employee from another unit. The employee states that a close friend is a patient on the nurse's unit. She asks how the friend is doing and if all her tests were normal. How should the nurse respond? 1. Answer the employee's questions softly so other people will not hear. 2. Decline to discuss her friend's condition and suggest that she visit her friend. 3. Give the employee the name of the patient's physician and suggest that she call for this information. 4. Tell the employee about the results of the patient's tests only if they were within normal limits.

2. Decline to discuss her friend's condition and suggest that she visit her friend. Rationale: All patient health information in this situation is confidential, regardless of the relationship of the employee to the patient. Therefore declining to discuss this information and suggesting visiting the patient is the best response. It is especially important that answering any questions regarding the patient's status or test results, including names and room number, not be discussed in public places to maintain the patient's right to confidentiality. The patient's physician should also follow the same procedure regarding discussion of a patient.

The nurse understands that the action of antidiuretic hormone (ADH) is to: 1. Reduce blood volume. 2. Decrease water loss in urine. 3. Increase urine output. 4. Initiate the thirst mechanism.

2. Decrease water loss in urine. Rationale: ADH is released by the posterior pituitary gland. It is mainly released in response to a decrease in blood volume, or an increased concentration of sodium or other substances in plasma. It acts to decrease the production of urine by increasing the reabsorption of water by renal tubules. A decrease in ADH would cause reduced blood fluid volume, decreased ability of the kidneys to reabsorb water resulting in increased urine output, and an increase in the thirst mechanism.

A patient has been admitted with a diagnosis of intractable vomiting and has only been able to tolerate sips of water. The initial blood work shows a sodium level of 122 mEq/L and a potassium level of 3.6 mEq/L. Based on the lab results and symptoms, what is the patient experiencing? 1. Hypenatremia 2. Hyponatremia 3. Hyperkalemia 4. Hypokalemia

2. Hyponatremia Rationale: The normal range for serum sodium is 135-145 mEq/L, and for serum potassium it is 3.5-5 mEq/L. Vomiting and use of diuretics, such as furosemide (Lasix), deplete the body of sodium. Without intervention, symptoms of hyponatremia may progress to include neurological symptoms such as confusion, lethargy, seizures, and coma. Hypernatremia results when serum sodium is greater than 145 mEq/L; hyperkalemia results when serum potassium is greater than 5.0 mEq/L; hypokalemia results when serum potassium is less than 3.5 mEq/L.

A patient presents to an urgent care center after sustaining a muscle sprain of the left ankle. Upon examination the nurse notes a developing hematoma, edema, and complaints of pain. The nurse should plan to apply which of the following? 1. Binder 2. Ice bag 2. Elastic bandage 3. Warm compress

2. Ice bag Rationale: Application of ice directly to a soft tissue injury causes vasoconstriction, which results in decreasing hemorrhage, edema, and pain. Use of a binder or elastic bandage to the area of a soft tissue injury is contraindicated and may cause compartment syndrome (constriction resulting in decreased circulation and nerve function). A warm compress would result in vasodilation and cause increased hemorrhage (hematoma formation), edema, and pain.

A patient with COPD reports steady weight loss and that he is too tired to eat. Which nursing diagnosis would be most appropriate for this patient? 1. Fatigue related to weight loss secondary to COPD 2. Imbalanced nutrition: less than body requirements, related to fatigue 3. Imbalanced nutrition: less than body requirements, related to COPD 4. Ineffective breathing pattern, related to alveolar hypoventilation

2. Imbalanced nutrition: Less than body requirements, related to fatigue. Rationale: The response portion of the nursing diagnosis is Imbalanced Nutrition: less than body requirements, and the etiology is fatigue associated with the diseases process of COPD. Interventions should be planned to deal with the breathing problem and the fatigue associated with it while implementing actions to combat the weight loss. Weight loss related to COPD is not a NANDA approved nursing diagnosis. Fatigue associated with the COPD disease process is the cause of the weight loss, not COPD in itself. Altered breathing pattern is also a problem but does not specifically relate to the weight loss problem.

When suctioning a patient with a tracheostomy, what is the most important safety measure the nurse must remember? 1. Hyperventilate the patient with room air prior to suctioning. 2. Initiate suction only as the catheter is being withdrawn. 3. Insert the catheter until the cough reflex is stimulated. 4. Remove the inner cannula before inserting the suction catheter.

2. Initiate suction only as the catheter is being withdrawn. Rationale: Use of suction upon withdrawal of a suction catheter reduces unnecessary removal of O2. In addition, suction should be applied intermittently during the withdrawal procedure to prevent hypoxia. A sterile catheter is used to prevent infection, and the catheter should only be inserted approximately 1 to 2 cm past the end of the trach tube to prevent tissue trauma. Hyperventilating a patient before suctioning should always be when O2, not room air. Inserting the catheter until the cough reflex is stimulated frequently occurs and does help to mobilize secretions but is not a safety measure. Removal of the inner cannula before inserting the suction catheter is not necessary.

A patient with a stage III pressure ulcer would benefit from which nursing intervention to prevent further injury by eliminating shearing force? 1. Maintaining the head of the bed at 35 degrees or lower. 2. Lifting the patient up in bed using a drawsheet with the help of another staff member. 3. Repositioning the patient at least every 2 hours and supporting her with pillows. 4. Performing passive ROM exercises of all extremities at least once per shift.

2. Lifting the patient up in bed using a drawsheet with the help of another staff member. Rationale: Shearing force is the pressure exerted on the skin when a debilitated patient is pulled up in bed without a drawsheet, or when the patient slides down in bed. With shearing, the skin adheres to the bed linens while the layers of subcutaneous tissue and bone slide in the direction of the body movements, causing a tearing of the skin. Using a drawsheet can reduce and minimize friction and shearing force. Answer options 1, 3, and 4 are all appropriate interventions to prevent further pressure injury and to promote circulation, but they are not as effective as option 2 in prevention of shearing force.

A patient with hyperthyroidism has been treated with radioactive iodine to destroy overactive thyroid gland cells. Which intervention is best for the nurse to use to reduce radiation exposure? 1. Wear a lead-shield apron. 2. Limit distance and time spent with the patient. 3. Wear a radiation meter to measure exposure. 4. Remain at least 6 feet away from the patient at all times.

2. Limit distance and time spent with the patient. Rationale: When caring for patients who are radioactive, the three most important concepts for reducing radiation exposure are to limit exposure time, increase distance, and use shielding. In this situation, time and distance provide the best reduction in radiation exposure. Wearing a lead-shield apron will help prevent radiation exposure, but time and distance are the first priorities. A radiation meter measures exposure but does nothing to protect caretakers. Remaining at least 6 feet away from the patient at all times is not a practical approach.

A nurse is preparing to administer an oil-retention enema and understands that it works primarily by: 1. Stimulating the urge to defecate. 2. Lubricating the sigmoid colon and rectum. 3. Dissolving the feces. 4. Softening the feces.

2. Lubricating the sigmoid colon and rectum. Rationale: The primary purpose of an oil-retention enema is to lubricate the sigmoid colon and rectum. Secondary benefits of an oil-retention enema include stimulating the urge to defecate and softening feces. An oil-retention enema does not dissolve feces.

A nurse is teaching a patient regarding the correct method to use for walking with crutches. The nurse should explain that weight must be placed: 1. In the axillae. 2. On the hands. 3. On the affected side. 4. On the unaffected side.

2. On the hands. Rationale: Body weight should be placed on the hands and not under the arms in the axillae when a patient is walking with crutches to prevent damage to the brachial plexus nerves and prevent "crutch paralysis". Placing weight in the axillae during crutch walking is incorrect. Weight during walking with two crutches should be distributed equally to both sides of the body without regard to the affected or unaffected sides.

When monitoring fluids and electrolytes, the nurse should have an understanding that the major cation-regulating intracellular osmolarity is: 1. Sodium 2. Potassium 3. Calcium 4. Calcitonin

2. Potassium Rationale: A decrease in serum potassium causes a decrease in the cell wall pressure gradient and results in water to move out of the cell. Besides intracellular osmolarity regulation, potassium also regulates metabolic activities, transmission and conduction of nerve impulses, cardiac conduction, and smooth and skeletal muscle contraction. Sodium is the most abundant extracellular cation that regulates serum osmolarity, as well as nerve impulse transmission and acid-base balance. Calcium is an extracellular cation necessary for bone and teeth formation, blood clotting, hormone secretion, cardiac conduction, transmission of nerve impulses, and muscle contraction. Calcitonin is a hormone secreted by the thyroid gland and works opposite of parathormone to reduce serum calcium and keep calcium in the bones. Calcitonin does not have a direct effect on intracellular osmolarity.

A nurse observes a colleague preparing a medication for IV bolus administration. Which medication being prepared should prompt the nurse to immediately intervene? 1. Saline flush 2. Potassium chloride 3. Naloxone (Narcan) 4. Adenosine (Adenocard)

2. Potassium chloride Rationale: Potassium chloride given as an IV bolus can cause cardiac arrest. It should never be administered intravenously without being diluted and infused slowly through an IV infusion pump. Saline flush, naloxone (Narcan), and adenosine (Adenocard) are appropriate to be given as IV bolus undiluted.

Which of the following would be an inappropriate use of a restraint device? 1. Preventing a confused patient from pulling out an IV. 2. Preventing an older adult patient from getting up at night due to a nursing staff shortage. 3. Maintaining immobilization of a patient's leg to prevent dislodging a skin graft. 4. Preventing an older adult patient from falling out of bed after a surgical procedure.

2. Preventing an older adult patient from getting up at night due to a nursing staff shortage. Rationale: Restraints are not used for staff convenience. An older adult who is unable to sleep should be assessed for physiological reasons for being unable to sleep and for safety needs before consideration of any restraint device. Various forms of restraint devices are indicated for patient protection from injury and to maintain essential medical therapies, such as pulling out an IV, dislodging a skin graft, or preventing falls.

In all states of the U.S. what is the professional nurse's legal responsibility regarding child abuse? 1. Honor the request of the parents not to report the suspected abuse. 2. Report any suspected abuse to local law enforcement authorities. 3. Return the child to the legal parent even if he or she is suspected of abuse. 4. Provide the parents with a copy of the child's medical record.

2. Report any suspected abuse to local law enforcement authorities. Rationale: In all states in the U.S. nurses and physicians are legally responsible for reporting suspected or actual abuse. Child protective services may obtain a court order to grant temporary guardianship if it is found that the child is in immediate danger. The other options (1,3,4) do not fulfill the nurse duty to report suspected child abuse.

The nurse is caring for a patient who is hyperventilating; the nurse knows this places the patient at risk for which of the following disorders? 1. Respiratory acidosis 2. Respiratory alkalosis 3. Respiratory compensation 4. Respiratory decompensation

2. Respiratory alkalosis Rationale: Hyperventilation causes excess amounts of carbon dioxide (CO2) to be eliminated, causing respiratory alkalosis. Respiratory acidosis is caused by excess CO2 retained in the lungs from conditions such as hypoventilation or COPD. Respiratory compensation and decompensation are terms not associated with this situation.

A nurse witnesses a patient climbing over the side rails and falling out of bed to the floor. Restraints has been ordered for the patient but were not in place. When the nurse completes the incident report, what information should the nurse include? 1. The fact that the nursing staff was not at fault because the patient initiated the accident. 2. The facts of the incident witnessed by the nurse as it occurred. 3. The name of the nurse who was responsible for monitoring the restraints. 4. The reason why the ordered restraints were not on the patient.

2. The facts of the incident witnessed by the nurse as it occurred. Rationale: The nurse filling out an incident report needs to state only the objective facts surrounding the incident, no opinion or speculation. In an incident report, fault or blame is subjective and should not be implied. It is not necessary to include names except for those of witnesses. Speculations or opinions as to the reason why the ordered restraints were not on the client are subjective and not appropriate to include in an incident or variance report.

A nurse places a heating pad on a patient and after 15 minutes realizes that the patient has been burned because the settings were incorrect. Which principle would apply? 1. No one could be held liable for new equipment. 2. The nurse could be held liable for the injury that occurred. 3. The nurse did what a reasonable, prudent nurse would do. 4. The manufacturer is liable for new equipment.

2. The nurse could be held liable for the injury that occurred. Rationale: A nurse can be held liable for any action performed that causes a client harm. Legally, someone will assume liability for the action. If sued in this case, the nurse would have to prove that her actions were reasonable and prudent under the circumstances. The manufacturer may also be held liable depending on whether the equipment was used correctly, but initially the actions of the nurse will be reviewed.

A 62-year-old male patient is being discharged home from the hospital. During his stay, he acquired a nosocomial infection, C.diff. In preparing a teaching plan for the patient and caretaker, which priority point would the nurse include? 1. Report any constipation to your physician immediately. 2. This infection causes diarrhea accompanied by flatus and abdominal discomfort. 3. The patient should consume a diet high in fiber and low in fat. 4. No special cleaning or disinfection will be required in the home.

2. This infection causes diarrhea accompanied by flatus and abdominal discomfort. Rationale: The main clinical manifestation of C.diff is diarrhea accompanied by excessive flatus and abdominal discomfort. Constipation is not associated with this infectious disease. Patients should follow a nutritionally balanced diet high in fiber and low in fats with no specific restrictions. Cleaning and disinfection of items in the home is key to preventing spread of the infection because the C.diff spore is relatively resistant.

The nurse is assessing a 56-year-old male patient who presents to the clinic for a routine physical and receives instruction about the need to obtain a stool specimen for occult blood testing (guiac test). The primary reason for this test is due to: 1. The client's age and gender. 2. A family history of polyps. 3. A routine examination for colon cancer. 4. A positive finding after a digital rectal exam.

3. A routine examination for colon cancer Rationale: The primary reason for a stool specimen for guiac occult blood testing is that it is part of a routine examination for colon cancer in any patient over the age of 40. Age and gender, family history of polyps, and a positive finding after a digital rectal exam are factors related to colon cancer and secondary reasons for the occult blood test (guiac test).

A nurse speaking in support of the best interest of a vulnerable patient reflects the nurse's duty of: 1. Caring 2. Veracity 3. Advocacy 4. Confidentiality

3. Advocacy Rationale: The nurse has a professional duty to advocate for a patient by promoting what is best for the patient. This is accomplished by ensuring that the patient's needs are met and by protecting the patient's rights. Caring is a behavioral characteristic of the nurse. Veracity relates to the habitual observance of truth, fact, and accuracy. Confidentiality is an ethical principle and legal right that the nurse will hold secret all information relating to the patient unless the patient gives consent to permit disclosure.

The nurse instructing an adolescent patient states that in addition to building bones and teeth, calcium is also important for: 1. Bile production. 2. Blood circulation. 3. Blood clotting. 4. Digestion of fats.

3. Blood clotting. Rationale: Calcium is important for blood coagulation. When tissue damage occurs, serum calcium is necessary to promote coagulation by activating certain clotting factors. Calcium acts as a catalyst in the clotting process in both the extrinsic and intrinsic pathways. Calcium is responsible for a number of bodily functions such as bone health, blood clotting, and muscle contraction and nerve impulses; however it is not directly related to bile and blood production or digestion of fats.

A patient is diagnosed with AIDS. The nurse recognizes that an opportunistic infection is present when the oral cavity is examined and white plaques are discovered on the mucosa. What does this finding most likely represent? 1. Cytomegalovirus 2. Histoplasmosis 3. Candida albicans 4. Human papillomavirus

3. Candida albicans Rationale: White patchy plaques on the oral mucosa would most likely be a result of Candida albicans, a yeastlike fungal infection. This condition is also known as thrush. Cytomegalovirus (1) may cause a serious viral infection in persons with HIV, resulting in retinal, gastrointestinal, and pulmonary manifestations. Histoplasmosis (2) is an infection caused by inhalation of spores of the fungus Histoplasma capsulatum and is characterized by fever, malaise, cough, and lymphadenopathy. Human papillomavirus (4) typically manifests as warts on the hands and feet as well as mucous membrane lesions of the oral, anal, and genital cavities. It may be transmitted without the presence of warts through bodily fluids with some forms associated with cancerous and precancerous conditions.

A bite from a large dog would cause which type of injury? 1. Abrasion 2. Fracture 3. Crush injury 4. Incisional laceration

3. Crush injury Rationale: The bite of a large dog can exert between 150 and 400 psi of pressure, causing a crush injury. A crush injury may or may not include a fracture. Abrasions and incisional lacerations are not caused by this form of trauma.

A nurse has admitted to the labor and delivery unit a patient who is 34 weeks' gestation. After discovering that the patient has been using heroin, what is the most appropriate action for the nurse? 1. Notify the nurse manager of the unit. 2. Inform no one because all patient information is confidential. 3. Inform the physician who will deliver the infant. 4. Alert the hospital security department because heroin is an illegal substance.

3. Inform the physician who will deliver the infant. Rationale: The fetus of a heroin-addicted mother is at risk for serious complications such as hypoxia and meconium aspiration. It is important to notify the physician of the patient's heroin use, because this information will influence the care of the patient and newborn. This information is used only in relation to the patient's care. With the patient's consent, it may be shared with other social service or health agencies that become involved with the patient's long-term care. The nurse manager of the unit may be notified as it relates to the care of the patient and her newborn. Patient information is confidential and only staff should be privileged to such information. Hospital security would only be notified if illicit substances were discovered on hospital premises.

The physician has declared a patient to be "brain dead". The nurse understands that this means that the client has: 1. No reflexes and no breathing. 2. Slow reflexes and shallow breathing. 3. No cortical functioning with some reflex breathing. 4. Deep tendon reflexes only and no independent breathing.

3. No cortical functioning with some reflex breathing. Rationale: A client who is declared as being brain dead has no function of the cerebral cortex and a flat EEG. The client may have some spontaneous breathing and a heartbeat. The guidelines established by the American Association of Neurology include coma or unresponsiveness, absence of brainstem reflexes, and apnea. There are specific assessments to validate the findings. The other answer options do not fit the definition of "brain dead".

A nurse receives abnormal results of a diagnostic testing. What is the nurse's ethical and legal accountability? 1. Inform the patient of the results. 2. Ensure that the results are placed in the patient's medical record. 3. Notify the patient's physician with the results. 4. Obtain normal values of the results from the lab.

3. Notify the patient's physician with the results Rationale: The nurse is most ethically and legally accountable for reporting diagnostic testing results to the patient's physician, whether the results are normal or, more important, abnormal. Informing the patient of the results is an incorrect action in this situation. Placing the results in the patient's record and obtaining normal values of the results from the lab are acceptable results for the nurse after notifying the physician of the abnormal results.

A 45-year-old male patient with a diagnosis of uncontrolled diabetes has been on the nursing unit for 5 days. Two days ago he was started on a diuretic. The patient's morning potassium level is 2.8 mEq/L. What is the most appropriate action for the nurse to take? 1. Hold the morning dose of the diuretic and have the lab repeat the test. 2. Do nothing different because the potassium is within normal limits. 3. Notify the physician with the results because the potassium is below the normal range. 4. Notify the physician with the results because the potassium is above the normal range.

3. Notify the physician with the results because the potassium is below the normal range. Rationale: The physician should be notified because a potassium level of 2.8 mEq/L is low. Normal range for serum potassium is 3.5-5 mEq/L. Patients who are on diuretics require monitoring of serum electrolytes, especially potassium and sodium, because they also are excreted with water. The nurse should not hold the diuretic or repeat the lab test unless advised by the physician. The patient's serum potassium level is critically below the normal limit and the physician should be notified (2,4).

Which physical assessment finding is most indicative of a systemic infection? 1. White blood cell (WBC) count of 8200/mm3 2. Bilateral 3+ pitting pedal edema 3. Oral temperature of 101.3 degrees F 4. Pale skin and nailbed color

3. Oral temperature of 101.3 degrees F Rationale: An elevated temperature of 101.3 degrees F is most indicative of a systemic infection. A white blood cell (WBC) count of 8200/mm3 is within the WBC normal range of 5000-10,000 mm3. Pedal edema is generally not related to an infectious process. Pale skin and nailbeds may be related to an infectious process but not necessarily.

A patient arrives at the health care provider's office complaining of several palpable elevated masses on his arms. Which term would most accurately describe these masses? 1. Erosion 2. Macule 3. Papule 4. Vesicle

3. Papule Rationale: Papules are superficial and elevated up to 0.5 cm. Nodules and tumors are masses similar to papules but are elevated more than 0.5 cm and may infiltrate deeper into tissues. Erosions are characterized as loss of the epidermis layer; macules are nonpalpable, flat changes in skin color less than 1 cm in diameter; and vesicles are usually transparent, filled with serous fluid, and are a blisterlike elevation.

A nurse assesses for hypocalcemia in a postoperative patient. One of the initial signs that might be present is: 1. Headache 2. Pallor 3. Paresthesias 4. Blurred vision

3. Paresthesias Rationale: Normally, calcium ions block the movement of sodium into cells. When calcium is low, this allows sodium to move freely into cells, creating increased excitability of the nervous system. Initial symptoms are paresthesias. This can lead to tetany if untreated. Headache, pallor, and blurred vision are not signs of hypocalcemia.

When a patient files a lawsuit against a nurse for malpractice, it is the patient who must prove there is a link between the harm suffered and what the nurse did negligently. What is this known as? 1. Evidence 2. Tort discovery 3. Proximate cause 4. Common cause

3. Proximate cause Rationale: Proximate cause is the legal concept meaning that the patient must prove that the nurse's actions contributed to or caused the patient's injury. Evidence is data presented in proof of the facts, which may include witness testimony, records, documents, or objects. A tort is a wrongful act, not including a breach of contract of trust that results in injury to another person. Common cause means to unite one's interest with another's.

A patient is ordered to receive morphine via patient-controlled analgesia (PCA). Before beginning administration of this medication, what should the nurse assess first? 1. Temperature 2. Neurological status 3. Respirations 4. Urinary output

3. Respirations Rationale: The nurse must be especially alert to any changes in respirations, because morphine decreases the respiratory center function in the brain. An order for morphine should be questioned if the baseline respirations are less than 12 per minute. Neurological status along with pulse and BP would be a priority assessment after respiratory rate. Measurements of temperature and urinary output are part of the overall client assessment but not a priority with morphine.

Which instruction should be included when teaching a patient about back safety? 1. Sleep on your side, and carry objects at arm's length. 2. Sleep on your back, and carry objects at arm's length. 3. Sleep on your side, and carry objects close to your body. 4. Sleep on your back, and carry objects close to your body.

3. Sleep on your side, and carry objects close to your body. Rationale: By sleeping on the side and carrying objects close to the center of the body, the client can lessen back strain. Sleeping on the back (2, 4) and carrying objects at arm's length (1, 2) add pressure and strain to the back muscles.

Which activity would be best for the nurse to take in preventing septic shock in the hospitalized patient? 1. Maintaining the patient in a normothermic state. 2. Administering blood products to replace fluid losses. 3. Using aseptic technique during all invasive procedures. 4. Keeping the critically ill patient immobilized to reduce metabolic demands.

3. Using aseptic technique during all invasive procedures. Rationale: Septic shock occurs as a result of an uncontrolled infection, which may be prevented by using correct infection control practices. These include aseptic technique during all invasive procedures. Maintaining the patient in a normothermic state, administering blood products, and keeping the patient immobilized are not directly related to the prevention of septic shock.

Which of the following legal defenses is the most important for a nurse to develop? 1. Dedication 2. Certification 3. Assertiveness 4. Accountability

4. Accountability Rationale: The concept of accountability is of high priority in nursing practice. As a licensed professional, the nurse is always accountable, which means liable and answerable for his or her actions. Dedication means to be committed and assertiveness means to be confident. These are desired characteristics in a nurse but not legal defenses. Certification relates to achieving a higher level of knowledge or proficiency in one's area of specialization and is also not a legal defense.

A nurse applies a cold pack to treat an acute musculoskeletal injury. Cold therapy decreases pain by which of the following actions? 1. Promotes analgesia and circulation. 2. Numbs the nerves and dilates the blood vessels. 3. Promotes circulation and reduction of muscle spasms. 4. Causes local vasoconstriction, prevents edema and muscle spasm.

4. Causes local vasoconstriction, prevents edema and muscle spasm. Rationale: Cold causes the blood vessels to constrict, which reduces the leakage of fluid into the tissues and prevents swelling and therefore muscle spasm. Cold therapy also may numb the nerves and surrounding tissues, thus reducing pain. Cold does promote analgesia but not circulation. It may numb nerves but does not dilate blood vessels.

A nurse preparing to apply restraints to a patient should understand which of the following principles? 1. The law prohibits restraining patients until a written order is obtained. 2. Charges of felony may be leveled against nurses who use restraints improperly. 3. Nurses are not obligated to report institutions that use restraints unlawfully. 4. Charges of assault and battery may be leveled against nurses who use restraints improperly.

4. Charges of assault and battery may be leveled against nurses who use restraints improperly. Rationale: Restraint of a patient, whether physical or chemical, is considered a high-risk procedure requiring a valid physician's order and intensive monitoring for safety and meeting the patient's needs. A nurse who does not follow correct procedures regarding restraints can legally be charged with assault and battery. Laws regarding restraint orders may differ from state to state and in different settings. A felony is a severe offense or crime such as murder, rape, or burglary and is commonly punished by imprisonment. Nurses have a professional obligation to report institutional misuse of restraints since this may constitute false imprisonment and abuse.

While performing a physical assessment on an immobilized patient, the nurse notes that a patient has shortened muscles over a joint, preventing full extension. What is this condition knowing as? 1. Osteoarthritis 2. Osteoporosis 3. Muscle atrophy 4. Contracture

4. Contracture Rationale: Immobilized patients are at a high risk for the development of contractures. Contractures are characterized by permanent shortening of the muscle covering a joint. Osteoarthritis is a disease process of the weight-bearing joints due to wear and tear. Osteoporosis is a metabolic disease process where the bones lose calcium. Muscle atrophy is a wasting and/or decrease in the strength and size of the muscles due to a lack of physical activity or a neurological or musculoskeletal disorder.

A nurse informs her patient that he should give someone the authority to make medical decisions for him in the event that he is unable to do so. What is the specific document that allows for this? 1. Advance directive 2. Living will 3. Client rights proxy 4. Durable power of attorney for health care

4. Durable power of attorney for health care Rationale: A durable power of attorney for health care is a document that authorizes the patient to name the person who will make the day-to-day and end-of-life decisions when the patient becomes unable to do so. A durable power of attorney is part of an advance directive or living will document. An advance directive, also sometimes called a living will, is a legal document in which the patient asks not to be kept alive by extraordinary medical effort when unable to make a decision. The document allows a patient to make his or her wishes known through execution of a formal, legally binding document. Client rights proxy is not a term associated with medical decisions.

Which of the following nursing diagnoses is correct for a nonambulatory patient with a reddened sacrum that is unrelieved by repositioning? 1. Risk for pressure ulcer. 2. Risk for impaired skin integrity. 3. Impaired skin integrity, related to infrequent turning and repositioning. 4. Impaired skin integrity, related to the effects of pressure and shearing force.

4. Impaired skin integrity, related to the effects of pressure and shearing force. Rationale: The impaired skin integrity is physiologically a result of unrelieved pressure and shearing force. This is supported by the data provided that the patient is nonambulatory and has a reddened sacrum. Risk for pressure ulcer is not an approved NANDA-I nursing diagnosis. The patient's problem is not being "at risk" because the patient already has an actual problem. Not enough information is provided to make the assumption that the impaired skin integrity is related to infrequent turning and repositioning.

A patient is being admitted to a medical unit with a diagnosis of pulmonary tuberculosis. Which type of room should this client be assigned by the nurse? 1. Private room 2. Semiprivate room 3. Room with windows that can be opened 4. Negative airflow room

4. Negative airflow room Rationale: TB is an airborne contagious disease that is best contained in a negative airflow room. Negative airflow rooms are always private. A private room, semiprivate room, and a room with windows that can be opened are not appropriate for the standard of care for a patient diagnosed with TB. Additionally, opening windows would present a possible safety hazard in a patient's room.

A surgical patient develops a wound infection during hospitalization. How is this type of infection classified? 1. Primary 2. Secondary 3. Superinfection 4. Nosocomial

4. Nosocomial Rationale: A nosocomial infection is acquired in a health care setting. This is also referred to as a hospital-acquired infection. It is a result of poor infection control procedures such as failure to wash hands between patients. A primary infection is synonymous with initial infection. A secondary infection is made possible by a primary infection that lowers the host's resistance and causes an infection by another kind of organism. A superinfection is a new infection caused by an organism different from that which caused the initial infection. The microbe responsible is usually resistant to the treatment given for the initial infection.

The nurse understands that dietary teaching for a patient with a colostomy should be based on the knowledge that: 1. Liquids should be limited to 1 L per day. 2. Nondigestable fiber and fruits should be eliminated. 3. A formed stool will result from a constipating diet. 4. The diet should be adjusted to include foods that result in manageable stools.

4. The diet should be adjusted to include foods that result in manageable stools. Rationale: Each person will need to experiment with diet after a colostomy to determine what foods are best tolerated and also produce stools that are manageable depending on the type of colostomy. Liquids are typically not limited unless there is a specific reason such as cardiac or renal disease. Food high in fiber such as fruit should be included in the diet as tolerated. Depending on the type of colostomy and the diet, a formed stool is acceptable and does not indicate a constipating diet.

Which of the following is indicated when a nurse performs a neurovascular assessment? Select all that apply. 1. Orientation 2. Capillary refill 3. Pupillary response 4. Respiratory rate 5. Pulse and skin temperature 6. Movement and sensation

Correct answers are 2, 5, and 6: Capillary refill, pulse and skin sensation, and movement and sensation Rationale: A neurovascular assessment involves evaluation of nerve and blood supply to an extremity involved in an injury. The area involved may include an orthopedic and/or soft tissue injury. A correct neurovascular assessment should include evaluation of capillary refill, pulses, warmth and paresthesias, and movement and sensation. Orientation, pupillary response, and respiratory rate are components of a neurological assessment.

What are the best ways for a nurse to be protected legally? Select all that apply. 1. Follow hospital policy and procedure whenever possible. 2. Establish a therapeutic relationship with all patients and families. 3. Provide care within the parameters of the state's nurse practice act. 4. Carry at least $100,000 worth of liability insurance. 5. Document consistently and objectively. 6. Clearly document a client's nonadherence to the medical regimen.

Correct answers are 3,5, and 6. Rationale: Malpractice or negligence must be proven legally. If a nurse is providing the best possible care under the circumstances, and within the scope of nursing practice, it would be difficult to prove allegations. Following hospital policy and procedure and establishing good relationships with patients and families does help to reduce legal risk. Liability insurance protects the nurse if found guilty and a monetary award is made, but it does not reduce the possibility of litigation. Consistent, objective, and clear documentation also support practice within legal parameters.

The patient asks the nurse to recommend foods that might be included in a diet for diverticular disease. Which foods would be appropriate to include in the teaching plan? Select all that apply. 1. Whole grains 2. Cooked fruits and vegetables 3. Nuts and seeds 4. Lean red meats 5. Milk and eggs

Correct answers: 1, 2, and 5 Rationale: With diverticular disease the patient should avoid foods that may obstruct the diverticuli. Therefore the fiber should be digestible, such as whole grains and cooked fruits and vegetables. Milk and eggs have no fiber content but are good sources of protein. In clients with diverticular disease, nuts and seeds are contraindicated as they may be retained and cause inflammation and infection, which is known as diverticulitis. The patient should also decrease intake of fats and red meats.

Droplet precautions

Used for droplets larger than 5 mcg and being within 3 feet of the patient such as streptococcal pharyngitis, mumps, and influenza.

Airborne precautions

Used for infected droplets smaller than 5 mcg such as measles, chickenpox, or pulmonary TB.

When performing a postoperative assessment, which parameter would alert the nurse to a common side effect of epidural anesthesia? 1. Decreased blood pressure. 2. Increased oral temperature. 3. Diminished peripheral pulses. 4. Unequal bilateral breath sounds.

1. Decreased blood pressure. Rationale: The most important side effect to monitor in a patient who has received epidural anesthesia is hypotension due to autonomic nervous system blockade. Therefore, in the immediate postoperative recovery period, the blood pressure should be assessed frequently. Other side effects include bradycardia, nausea, and vomiting. Increased oral temperature (2) and unequal bilateral breath sounds (4) are not effects associated with epidural anesthesia. Diminished peripheral pulses (3) may result from hypotension, although they are not the most common side effects.

The nurse is assessing an edematous patient and is aware that edema occurs in which extracellular fluid compartment? 1. Interstitial 2. Intercellular 3. Intravascular 4. Intracellular

1. Interstitial Rationale: Edema is defined as the accumulation of fluid in the interstitial spaces. The incorrect answer options occur in other compartments: intercellular means between or among cells; intravascular means within a vascular space; and intracellular means within a cell.

In assessing fluid and electrolyte status, the nurse recognizes that the regulator of extracellular osmolarity is: 1. Sodium 2. Potassium 3. Chloride 4. Calcium

1. Sodium Rationale: Sodium is the most abundant extracellular fluid cation and regulates serum (extracellular) osmolarity as well as nerve impulse transmission and acid-base balance. Potassium is the major intracellular osmolarity regulator, and it also regulates metabolic activities, transmission and conduction of nerve impulses, cardiac conduction, and smooth and skeletal muscle contraction. Chloride is a major extracellular fluid anion and follows sodium. Calcium is an extracellular cation necessary for bone and teeth formation, blood clotting, hormone secretion, cardiac conduction, transmission of nerve impulses, and muscle contraction.

A patient has returned from surgery with a tracheostomy tube in place. After about 10 minutes in postoperative recovery, the patient begins to have noisy, increased respirations and an elevated heart rate. What action should the nurse take immediately? 1. Suction the tracheostomy. 2. Change the tracheostomy tube. 3. Readjust the tracheostomy tube and tighten the ties. 4. Perform a complete respiratory assessment.

1. Suction the tracheostomy tube. Rationale: Noisy, increased respirations and increased pulse are signs that the patient needs immediate suctioning to clear the airway of secretions. After suctioning, a complete respiratory assessment should be performed. After suctioning, then performing a respiratory assessment (4), further problem solving may require readjust of tracheostomy tube and ties (3) or a physician changing the tracheostomy tube (2).

A patient is scheduled for a transurethral resection of the prostate (TURP). Which statement made by the patient most indicates the need for further preoperative teaching? 1. "My urine will be red after surgery." 2. "I will have a catheter after surgery." 3. "My incision will probably be painful." 4. "I will need to drink a lot after surgery."

3. "My incision will probably be painful." Rationale: The TURP procedure is performed by insertion of a scope device into the urethra to reach the prostate from within the urinary tract. No incision is made to reach the prostate, therefore the patient statement about an incision being painful after surgery warrants further evaluation and teaching by the nurse. The patient is demonstrating correct knowledge about the TURP procedure by stating that after surgery his urine will be red (1), he will have a catheter (2), and he will need to increase fluid intake (4).

A nurse auscultates a murmur at the 2nd left intercostal space (ICS) along the sternal border. This reflects sound from which valve? 1. Aortic 2. Mitral 3. Pulmonic 4. Tricuspid

3. Pulmonic Rationale: The 2nd left ICS along the sternal border reflects sound from the pulmonic valve. The correct landmark for auscultating the aortic valve is at the right 2nd ICS sternal border; for the mitral valve (apical pulse) at the left 5th ICS, midclavicular line; and for the tricuspid valve at the left 5th ICS sternal border.

Protective environment

Focuses on clients with a compromised immune system to protect them from incoming pathogens

TED stockings

Graduated compression that promotes venous return and decreases risk of VTE Priority: proper size, no folds/wrinkles, wounds are covered with dressings

When caring for a patient with a fractured hip, the nurse should place pillows around the injured leg to specifically maintain: 1. Abduction 2. Adduction 3. Traction 4. Elevation

1. Abduction Rationale: Abduction means to move the limb away from the median plane, or axis, of the body. In care of the client with a fractured hip, the legs and hip must be aligned in an abducted position position to prevent internal rotation, reduce the risk of dislocation, and decrease pain. In a patient with a fractured hip, adduction of the limb, traction, and elevation are not appropriate procedures. Adduction means to move the limbs toward the medial plane, or axis, of the body, and traction involves the process of applying a pulling force in opposite directions using weights.

A nurse is teaching a new nursing assistant about ways to prevent the spread of infection. Included in the instruction would be the fact that the cycle of the infectious process must be broken, which may be accomplished primarily through: 1. Handwashing before and between providing patient care 2. Cleaning all equipment with an approved disinfectant after use 3. Wearing infection control-approved protective equipment when providing patient care 4. Using medical and surgical aseptic techniques at all times

1. Handwashing before and between providing patient care Rationale: Handwashing before and after providing care is the single most effective means of preventing the spread of infection by breaking the cycle of infection. Although all these interventions are acceptable procedures and may assist in preventing the spread of infection, none are as effective as handwashing.

A patient is scheduled for a surgical resection of the colon and creation of a colostomy for a bowel malignancy. The nurse would anticipate the physician to order a prep with antibiotics before surgery primarily to: 1. Decrease peristalsis. 2. Minimize electrolyte imbalance. 3. Decrease bacteria in the intestines. 4. Treat inflammation caused by the malignancy.

3. Decrease bacteria in the intestines. Rationale: To decrease the possibility of contamination, the bacteria count in the colon is lowered with antibiotics before surgery. Preoperative antibiotics do not have an effect on peristalsis (1), electrolyte imbalance (2), or treating inflammation (4).

A nurse teaches a patient to apply antiembolism elastic stockings. What is the correct technique? 1. Apply at bedtime and remove before getting out of bed in the morning. 2. Apply in the morning before legs are lowered from the bed to the floor. 3. Roll down thigh-high stockings to the knees if the patient is immobile. 4. Apply in the morning or at bedtime only after the legs are lowered to the floor.

2. Apply in the morning before legs are lowered from the bed to the floor. Rationale: Applying antiembolism stockings in the morning before the legs are lowered to the floor prevents excessive blood from collecting and being trapped in the lower extremities as a result of the force of gravity. Elastic stockings are worn to prevent the formation of emboli and thrombi, especially in clients who have had surgery or who have limited mobility, by applying constant compression. It is contraindicated for antiembolism elastic stockings to be applied and worn at night, rolled down, or applied after the legs are lowered to the floor.

A physician plans to remove a chest tube on a patient. The nurse would anticipate which intervention before removal? 1. Assist the physician to disconnect the drainage system from the chest tube. 2. Ensure the results of the chest x-ray are available, revealing lung reexpansion. 3. Obtain an arterial blood gas to document oxygenation status. 4. Sedate the patient before the physician removes the chest tube.

2. Ensure the results of the chest x-ray are available, revealing lung reexpansion. Rationale: A chest x-ray should be performed to ensure and to document that the lung is reexpanded and has remained expanded. The drainage system should not be disconnected from the actual chest tube while still in the patient because this may cause a pneumothorax to recur. An arterial blood gas may be performed prior to removal but is not necessary. An oxygen saturation reading with a pulse oximeter is usually sufficient to determine oxygenation level. The patient may be given pain medication before the procedure but not sedation, as this may decrease the oxygen status.

A nurse is providing preoperative teaching for a patient who is scheduled for a cholecystectomy. Which postoperative routine should the nurse emphasize as initially most important? 1. Early ambulation. 2. Coughing and deep breathing. 3. Wearing antiembolic elastic stockings. 4. Maintenance of a nasogastric tube.

2. Coughing and deep breathing. Rationale: The patient who has a cholecystectomy will have difficulty taking deep breaths and coughing because of the location of the surgical incision. Therefore it is important to instruct the patient preoperatively to improve compliance with the procedure in the early post-op period. Although ambulation (1), antiembolism stockings (3), and maintaining a nasogastric tube (4), if ordered, are important postoperative procedures, maintaining the airway and prevention of further pulmonary problems is the priority.

A patient is receiving parenteral nutrition (TPN) through a subclavian triple-lumen catheter. Upon evaluation, the nurse discovers that the TPN bag is empty and the next bag has not been received from the pharmacy. What is the most appropriate action for the nurse to take? 1. Perform a fingerstick glucose test and call the physician with the results. 2. Hang a bag of 10% dextrose at the ordered TPN rate and call the pharmacy to expedite the order. 3. Discontinue the infusion and flush the IV line with saline solution until the next TPN bag is ready. 4. Hang a bag of 5% dextrose at a keep-open rate until the next TPN bag is ready.

2. Hang a bag of 10% dextrose at the ordered TPN rate and call the pharmacy to expedite the order. Rationale: Patients receiving TPN require monitoring of blood glucose because the TPN solution contains a high concentration of dextrose. In response to the high-dextrose TPN solution, the pancreas increases production of insulin to meet the glucose demands. In this situation, the current TPN infusion is completed, and the nurse should infuse 10% dextrose to compensate for the loss while the next TPN bag is being prepped. If this action is not taken, the patient could experience a profound hypoglycemic reaction. After beginning an infusion of 10% dextrose, the nurse may perform a fingerstick glucose test and notify the physician if the results are abnormal. Discontinuing the infusion and flushing the line until the next TPN bag is ready is not recommended. Starting an infusion of 5% dextrose at KVO until the next TPN is ready may not prevent hypoglycemia.

While undergoing a soapsuds enema, the patient complains of abdominal cramping. What is the best action for the nurse to take? 1. Immediately stop the infusion. 2. Lower the height of the enema bag. 3. Advance the enema tubing 2-3 inches. 4. Clamp the tube for 2 minutes, then restart the infusion.

2. Lower the height of the enema bag. Rationale: Abdominal cramping during a soapsuds enema may be due to too rapid administration of the enema solution. Lowering the height of the enema bag slows the flow and allows the bowel time to adapt to the distension without causing excessive discomfort. Stopping the infusion is not necessary. Advancing the enema tubing is not appropriate. Clamping the tube for several minutes then restarting the infusion may be attempted if slowing the infusion does not relieve the cramps.

A patient reports smoke coming from a utility room on the nursing unit. What is the initial action the nurse should take? 1. Pull the fire alarm on the unit. 2. Remove anyone in immediate danger. 3. Obtain a fire extinguisher and report to the fire area. 4. Close all windows and fire doors and await further instructions.

2. Remove anyone in immediate danger. Rationale: The nurse is following the standard fire safety procedure RACE: "R" represents removing any patients from immediate danger. "A" represents alarming or activating the fire alarm, "C" represents containing the fire source by closing all windows and fire doors, and "E" represents extinguishing the fire and/or evacuating.

A patient is receiving a unit of packed red blood cells (PRBC). The patient experiences tingling in the fingers and HA. What is the nurse's priority action? 1. Call the physician. 2. Stop the transfusion. 3. Slow the infusion rate. 4. Assess the IV site for infiltration.

2. Stop the transfusion. Rationale: Tingling in the fingers and HA may be an indication of an adverse reaction to the transfusion. The nurses' priority action is to stop the infusion and begin a normal saline infusion at KVO (keep vein open). The patient should be assessed-including vital signs-then the physician should be notified. Slowing of the infusion rate is not appropriate if the patient is experiencing a reaction or is suspected of having a reaction. Assessment of the IV site is part of the general patient assessment and is not related to a blood transfusion reaction.

A nurse is instructing a disabled patient regarding how to safely use a cane. The nurse should demonstrate proper use of the cane by holding it on: 1. Alternating sides. 2. The unaffected (stronger) side. 3. The affected (weaker) side. 4. The side of the patient's choice.

2. The unaffected (stronger) side. Rationale: The cane should be used on the stronger (unaffected) side of the body to add strength, decrease dependence on the weaker (affected) side, and aid in balance during ambulation. Correct use of a cane does not involve alternating sides, using the cane on the affected (weaker) side, or using the side of the patient's choice.

A nurse is teaching a spinal cord injured patient who is being prepared for discharge home the proper technique for intermittent urinary self-catheterization. Which instruction is most important for the nurse to include? 1. Wear sterile gloves when doing the procedure. 2. Wash your hands before performing the procedure. 3. Perform the catheterization on yourself every 12 hours. 4. Dispose of the catheter after you have done the catheterization.

2. Wash your hands before doing the procedure. Rationale: To avoid transferring organisms to the urinary system, the patient is taught to wash his or her hands thoroughly with soap and water before inserting a clean catheter. Sterile gloves are not required for this procedure in the home care setting. Every 12 hours is too long of a time frame between catheterizations. The patient should be taught to recognize when self-catheterization is needed and develop a 2-3 hour catheterization schedule. Some home care settings may require the patient to clean and re-use catheters.

A 16-year-old male patient had an inguinal hernia repaired in the day surgery unit and is being prepared for discharge home. The nurse instructs him and his parents about resumption of physical activities. Which statement would indicate that the patient understands the instructions? 1. "I can ride my bike in about a week." 2. "I don't have to go to gym class for 3 months." 3. "I may be able to play basketball when my doctor says it is okay." 4. "I can never participate in weightlifting competitions again."

3. "I may be able to play basketball when my doctor says it is okay." Rationale: By acknowledging that playing basketball is a restricted activity until cleared by the surgeon indicates the patient understands the discharge instructions. Activities such as bike riding (1), physical education classes (2), playing basketball, and weightlifting (4) are contraindicated for at least 3 weeks after uncomplicated surgery for an inguinal hernia. Refraining from these activities for this period of time prevents stress on the incision and promotes healing. However, the patient should not participate in any of these activities until cleared by the surgeon.

A patient is to receive a transfusion of packed red blood cells (PRBCs). The nurse should prepare for the transfusion by priming the blood IV tubing with which solution? 1. Ringer's lactate 2. 5% dextrose and water 3. 0.9% normal saline 4. 0.45% normal saline

3. 0.9% normal saline Rationale: Blood and blood products for transfusion should be infused/diluted only with 0.9% normal saline solution. Solutions other than normal saline are incompatible and may cause RBC destruction by hemolysis.

A nurse is assessing drainage on a surgical dressing. Which item of documentation is most informative regarding the findings? 1. Moderate amount of drainage. 2. No change in drainage since yesterday. 3. A 10-mm-diameter area of drainage at 1900 hours. 4. Drainage is doubled in size since last dressing change.

3. A 10-mm-diameter area of drainage at 1900 hours. Rationale: A 10-mm-diameter area of drainage at 1900 hours is objective data and gives specific details regarding the assessment and a time frame. By providing size, it establishes parameters to compare with previous assessments and to further evaluate the drainage. Answer options 1, 2, and 4 are not specific, objective, or measurable.

The postanesthesia care unit (PACU) has just called in the report on a patient who has undergone a subtotal thyroidectomy and will be returning to the nursing unit. What emergency equipment is most important for the nurse to have available for this patient? 1. A defibrillator 2. An IV infusion pump 3. A tracheostomy tray 4. An electrocardiogram (ECG) monitor

3. A tracheostomy tray Rationale: The patient who has undergone a subtotal thyroidectomy is at high risk for airway occlusion resulting from postoperative edema. With this is mind, emergency airway equipment such as a tracheostomy set and intubation supplies should be immediately available to the patient. A defibrillator (1), an IV infusion pump (2), and an electrocardiogram (ECG) monitor (4) are all equipment items that should be available to all postoperative patients.

A patient's chest tube has accidentally dislodged. What is the nursing action of highest priority? 1. Lay the patient down on the left side. 2. Lay the patient down on the right side. 3. Apply a petroleum gauze dressing over the site. 4. Prepare to reinsert a new chest tube.

3. Apply a petroleum gauze dressing over the site. Rationale: A petroleum gauze dressing will prevent air from being sucked into the pleural space, causing a pneumothorax. The petroleum gauze dressing should be taped only on 3 sides to allow for excessive air to escape, preventing a tension pneumothorax. The physician should be immediately notified and the patient assessed for signs of respiratory distress. Preparing to reinsert a new chest tube is not a priority of the nurse at this moment. Positioning the patient on either the left or right side will not make a difference in outcome.

A nurse assisting in a research study calculates the risk-benefit ratio and concludes that there were no harmful effects associated with a survey of diabetic clients. This researcher was applying which principle? 1. Human dignity 2. Human rights 3. Beneficence 4. Utilitarianism

3. Beneficence Rationale: Beneficence is defined as the promotion of well-being and abstaining from the injuring of others as well as doing good, being kind, and charitable. In this situation, the possible benefits outweigh the possible harm for the clients participating in a research study. In this situation, human dignity and human rights are underlying principles of research ethics but are not directly related to the risk benefit ratio. Utilitarianism relates to the ethical doctrine that virtue is based on utility and that conduct should be directed toward promoting the greatest good for the greatest number of people.

The nurse should instruct a patient with an ileal conduit to empty the collection device frequently because it may: 1. Force urine to back up into the kidneys. 2. Suppress production of urine. 3. Cause the device to pull away from the skin. 4. Tear the ileal conduit.

3. Cause the device to pull away from the skin. Rationale: If the device becomes full and is not emptied, it may pull away from the skin and leak urine. Urine in contact with unprotected skin will irritate and cause skin breakdown. A full urine collection bag will not cause urine to back up into the kidneys, suppress the production of urine, or tear the ileal conduit.

A nurse working in the Emergency Department is assessing a 79-year-old patient who arrived via ambulance. Which of the following, if found during assessment, is considered an early sign of dehydration? 1. Sunken eyes 2. Dry, flaky skin 3. Change in mental status 4. Decreased bowel sounds

3. Change in mental status Rationale: Older adults are sensitive to changes in fluid and electrolyte levels, especially sodium, potassium, and chloride. These changes will manifest as a change in mental status and confusion. It is difficult to assess dehydration in older adults based on sunken eyes, dry skin, and decreased bowel sounds, because these can be prominent as general normal findings in the elderly patient.

A patient is receiving fresh frozen plasma (FFP). The nurse would expect to see improvement in which condition? 1. Thrombocytopenia 2. Oxygen deficiency 3. Clotting factor deficiency 4. Low hemoglobin

3. Clotting factor deficiency Rationale: FFP is an unconcentrated form of blood plasma containing all of the clotting factors except platelets. It can be used to supplement red blood cells (RBCs) when other blood products are not available or to correct a bleeding problem of unknown cause. Thrombocytopenia is a condition of low platelet count and is not treated with FFP. An oxygen deficiency and low hemoglobin may be improved indirectly with FFP, but it is not a definitive treatment.

A nurse assesses the lungs of a patient and auscultates soft, crackling, bubbling breath sounds that are more obvious on inspiration. This assessment should be documented as: 1. Vesicular. 2. Bronchial. 3. Crackles. 4. Rhonchi.

3. Crackles Rationale: Crackles are abnormal breath sounds described as soft, crackling, bubbling sounds produced by air moving across fluid in the alveoli. Vesicular breath sounds (1) are normal. They are quiet, soft, and inspiration sounds that are short and almost silent on expiration. They are heard over the lung periphery. Bronchial breath sounds (2) are normal and consist of a full inspiration and expiratory phase with the expiratory phase being louder. They are heard over the trachea and large bronchi of the lungs. Rhonchi (4) are abnormal breath sounds heard over the large airways of the lungs. They consist of a low pitch and are caused by the movement of secretions in larger airways; they usually clear with coughing.

A nurse is caring for a postoperative patient who has a nasogastric tube set to low intermittent suction. An IV of 5% dextrose with 0.45% sodium chloride and 20 mEq of potassium is ordered to primarily prevent which condition? 1. Constipation 2. Dehydration 3. Electrolyte imbalance 4. Nausea and vomiting

3. Electrolyte imbalance Rationale: When patients do not receive nutrients or fluids by mouth, and have loss of electrolytes through the removal of gastric secretions via a nasogastric (NG) tube, then electrolyte imbalance is a primary concern. Although dehydration is a possible effect of an NG tube removing gastric secretions and fluid, electrolyte imbalance is still the priority. Constipation is usually not a concern in this type of situation. An NG tube set to low intermittent suction usually relieves nausea and vomiting.

Percussion may be performed on patients with respiratory problems. The primary purpose of this procedure is to: 1. Relieve bronchial spasm. 2. Increase depth of respirations. 3. Loosen pulmonary secretions. 4. Expel carbon dioxide from the lungs.

3. Loosen pulmonary secretions. Rationale: Percussion (chest physiotherapy) loosens pulmonary secretions by mechanical means. This is accomplished by vibrations over the lung fields on the patient's posterior, anterior, and lateral chest. Percussion does not relieve bronchial spasms. Once pulmonary secretions are loosened by percussion and the patient has a clearer airway, the depth of respirations may increase and facilitate removal of carbon dioxide from the lungs.

In the immediate postoperative period, which action is the most important for a nurse to include in a patient's plan of care to prevent thrombophlebitis? 1. Increase fluid intake. 2. Restrict fluids. 3. Encourage early mobility. 4. Elevate the knee gatch of the bed.

3. Encourage early mobility. Rationale: In the immediate postoperative period, mobility is encouraged because veins require the assistance of the surrounding muscle beds to help pump blood toward the heart. This reduces venous stasis and the risk of thrombophlebitis. Increased fluid intake (1), if not contraindicated, will prevent dehydration and venous stasis. Therefore, restriction of fluids (2) may promote venous stasis and increase risk. Elevating the knee gatch of the bed (4) will impede venous blood flow and also increase the risk for thrombophlebitis.

If a hospital threatens to retain a newborn until their parents pay part of their bill, which legal term best describes the situation? 1. False threats 2. Assault and battery 3. False imprisonment 4. Breach of confidentiality

3. False imprisonment Rationale: The hospital is threatening to keep the infant; therefore false imprisonment is threatened. False imprisonment is restraining or confining a person without a clinical reason. False threat (1) may be a term to describe false imprisonment; however it is inaccurate in this situation. Assault and battery (2) legally means to threaten violence and the physical act of violence. Breach of confidentiality (4) is a disclosure to a third party, without client consent or court order of private information.

The amount of hemoglobin in the blood has what effect on oxygenation status? 1. The effect is little to none. 2. More hemoglobin reduces respiratory rate. 3. Low hemoglobin levels cause reduced oxygen-carrying capacity. 4. Low hemoglobin levels cause increased oxygen-carrying capacity.

3. Low hemoglobin levels cause reduced oxygen-carrying capacity. Rationale: Hemoglobin carries oxygen to all tissues in the body. If the hemoglobin level is low, the amount of oxygen-carrying capacity is also low. Higher levels of hemoglobin will increase oxygen-carrying capacity and thus increase the total amount of oxygen available in the blood. Answer option 1 is an untrue statement. If a patient had an increased respiratory rate (compensatory reaction to increase oxygen intake and availability) due to low hemoglobin, more hemoglobin (2) may have an indirect effect to decrease the respiratory rate. Low hemoglobin results in decreased oxygen-carrying capacity (4).

A patient is on a low carbohydrate diet in which there is decreased glucose available for energy. With this type of diet, fat is metabolized for energy and results in an increased production of which substance in the urine? 1. Protein 2. Glucose 3. Ketones 4. Uric acid

3. Ketones Rationale: As a result of fat metabolism, ketone bodies are formed and the kidneys attempt to decrease the excess by filtration and excretion. Excessive ketones in the blood can cause metabolic acidosis. A low carbohydrate diet does not cause increased protein, glucose, or uric acid (1, 2, 4) in the urine.

A nurse fails to act in a reasonable, prudent manner. Which legal principle is most likely to be applied? 1. Malice 2. Tort law 3. Malpractice 4. Case law

3. Malpractice Rationale: Malpractice is the unskilled or faulty treatment by a professional that causes injury or harm to a patient. It can result from a lack of professional knowledge or skill that can be expected in others in the profession, or from a failure to exercise reasonable care or judgment in the application of professional knowledge, experience, or skill. Malice is the desire or intent to inflict injury, harm or suffering. Tort law is a wrongful act, not including a breach of contract of trust, that results in injury to another person and for which the injured person is entitled to compensation. Case law is law established by judicial decisions in particular cases instead of by legislation action.

A patient had a liver bipolar performed. The nursing action of highest priority to prevent postprocedure hemorrhage would be to place the patient: 1. Supine and flat in bed. 2. In a sitting position. 3. On the right side. 4. On the left side.

3. On the right side. Rationale: Placing a patient on the right side after a liver biopsy compresses the liver against the abdominal wall, thus holding pressure on the biopsy site and allowing clot formation. These three positions (1, 2, 4) are inappropriate for a patient after a liver biopsy.

A patient on a mechanical ventilator is receiving positive end-expiratory pressure (PEEP). The nurse understands this treatment improves oxygenation primarily by: 1. Providing more oxygen to lung tissue. 2. Adding pressure to lung tissue, which improves gas exchange. 3. Opening collapsed alveoli and maintaining them open. 4. Opening collapsed bronchioles, allowing more oxygen to reach lung tissue.

3. Opening collapsed alveoli and maintaining them open. Rationale: The primary mechanism of PEEP is to deliver positive pressure to the lung at the end of expiration. This helps to open collapsed alveoli and keep them open. With the primary mechanism of PEEP to open the alveoli and maintain them open, exchange of carbon dioxide and oxygen can take place more efficiently, thus improving oxygenation by providing more oxygen to the lung tissue (1) and improving gas exchange (2). Peep may have an indirect effect on opening bronchioles (4).

While the nurse moves a patient from a lying to standing position, the patient experiences a rapid drop in blood pressure. The nurse would report this finding to the physician as: 1. Malignant hypotension 2. Orthostatic dehydration 3. Orthostatic hypotension 4. Vasomotor instability

3. Orthostatic hypotension Rationale: Orthostatic hypotension specifically refers to an abnormally low blood pressure that occurs when an individual assumes a standing position. Orthostatic hypotension is also known as postural hypotension. It may be a result of internal bleeding, fluid depletion, or loss of neurovascular control preventing vasoconstriction to regulate blood pressure. Malignant hypotension and orthostatic dehydration are inaccurate terms not used. Vasomotor instability occurs during menopause and results in hot flashes and night sweats.

The nurse would assess a patient 24-48 hours postoperatively for which problem as a result of anesthetic agents? 1. Colitis 2. Stomatitis 3. Paralytic ileus 4. Gastrocolic reflux

3. Paralytic ileus Rationale: After surgery patients are at risk for paralytic ileus as a result of receiving an anesthetic agent. The nurse can prevent or minimize paralytic ileus by increasing movement as soon as possible after surgery, through actions such as turning and early ambulation. Evidence of bowel function returning to normal includes auscultation of bowel sounds and passing of flatus and stool. Colitis (1), stomatitis (2), and gastrocolic reflux (4) are not postoperative complications related to anesthetic agents.

A nurse is measuring a patient's vital signs and uses a pulse oximeter to obtain which of the following types of information? 1. Respiratory rate 2. Amount of oxygen in the blood 3. Percentage of hemoglobin-carrying oxygen 4. Amount of carbon dioxide in the blood

3. Percentage of hemoglobin-carrying oxygen Rationale: The pulse oximeter measures the oxygen saturation of blood by determining the percentage of hemoglobin-carrying oxygen. A pulse oximeter does not interpret the amount of oxygen or carbon dioxide carried in the blood, nor does it measure respiratory rate.

The nurse suspects that an intraoperative patient has a distended bladder. Which method is correct to assess for this condition? 1. Inspect and palpate in the epigastric region. 2. Auscultate and percuss in the inguinal region. 3. Percuss and palpate in the hypogastric region. 4. Percuss and palpate bilaterally in the lumbar areas.

3. Percuss and palpate in the hypogastric region. Rationale: To detect a distended bladder, percussion and palpation should be performed over the hypogastric region of the abdomen. Percussion of a distended bladder would produce a dull sound and feel firm on palpation. Answer options 1, 2, and 4 are all inaccurate procedures to assess for a distended bladder.

A high-protein diet is recommended for a patient recovering from a fracture. What is the rationale for a high-protein diet? 1. Promotes gluconeogenesis. 2. Has an antiinflammatory effect. 3. Promotes cell growth and bone union. 4. Decreases pain medication requirements.

3. Promotes cell growth and bone union. Rationale: There is an increased need for protein with any type of body tissue trauma. High protein intake in the patient with a fractured bone promotes cell growth and therefore bone union. Intake of a high protein diet during recovery from a bone fracture is not related to gluconeogenesis, inflammation, or pain.

A nurse should place the patient in which position to obtain the most accurate reading of jugular vein distension? 1. Upright at 90 degrees 2. Supine position 3. Raised to 30 degrees 4. Raised to 10 degrees

3. Raised to 30 degrees Rationale: Jugular vein pressure is measured with a centimeter ruler to obtain the vertical distance between the sternal angle and the point of highest pulsation. This procedure is most accurate when the head of the bed is elevated between 30 and 45 degrees. The internal and external jugular veins should be inspected while the patient is gradually elevated from a supine position to an upright 30-45 degrees. Jugular vein distension cannot accurately be assessed if the patient is supine, at 90 degrees, or 10 degrees.

A nurse is assessing a patient with a pressure ulcer and finds it to be full thickness with necrosis into the subcutaneous tissue down to the underlying fascia. These data would most describe which stage of pressure ulcers? 1. Stage I 2. Stage II 3. Stage III 4. Stage IV

3. Stage III Rationale: A pressure ulcer that is full thickness with necrosis and ulceration into the subcutaneous tissue and down to, but not through, the underlying fascia is characteristic of a stage III pressure ulcer. A stage I pressure ulcer is defined as an area of persistent redness with no break in skin integrity. A stage II pressure ulcer is a partial thickness wound with skin loss involving the epidermis, dermis, or both; the ulcer is superficial and may present as an abrasion, blister, or shallow crater. A stage IV pressure ulcer involves full thickness skin loss with extensive damage and tissue necrosis to muscles, support tissues, and bone; undermining and sinus tracts may also be present.

What would be the best nursing intervention to prevent footdrop in a leg with a cast? 1. Encourage complete bed rest. 2. Support the foot with 45 degrees of flexion. 3. Support the foot with 90 degrees of flexion. 4. Place an elastic stocking on the foot to provide support.

3. Support the foot with 90 degrees of flexion. Rationale: To prevent footdrop (plantar flexion of the foot due to weakness or paralysis of the anterior muscles of the lower leg) in a patient with a cast, the foot should be supported with 90 degrees of flexion. Bed rest can cause footdrop, and 45 degrees is not enough flexion to prevent footdrop. Application of an elastic stocking for support also will not prevent footdrop; a firmer support is required.

A 62-year-old female patient being prepared for discharge is ordered to go home with a walker. The nurse has instructed the patient regarding proper use of this assistive device. Which outcome is evidence that the instructions provided by the nurse were effective? 1. The patient picks up the walker and carries it for short distances. 2. The patient uses the walker only when someone else is present. 3. The patient moves the walker no more than 12 inches in front of her during use. 4. The patient states she will buy a walker on her way home from the hospital.

3. The patient moves the walker no more than 12 inches in front of her during use. Rationale: Safety is always a consideration when teaching a patient how to use an assistive device. Therefore, the correct procedure regarding using a walker is to move the walker no more than 12 inches in front to maintain balance and to be effective in forward movement. Carrying the walker when ambulating is incorrect. Once the patient is instructed and can demonstrate correct use of a walker, there is no need for someone to be present every time the patient uses the walker. If the patient is ordered to use a walker as part of the discharge plan, it needs to be provided before leaving the hospital.

A nurse is performing a postoperative assessment on a patient who has had surgery to correct an upper urinary tract obstruction. Which finding noted 8 hours after the surgery should the nurse report to the surgeon? 1. Incisional pain 2. Absent bowel sounds 3. Urine output of 20 mL/hour 4. Serosanguineous drainage on the dressing

3. Urine output of 20 mL/hour Rationale: A urinary output of 50 mL/hr or greater is necessary to prevent stasis and consequent infections after this type of surgery. The nurse should notify the surgeon of the assessment findings, since this may indicate a urinary tract obstruction. Incisional pain (1), absent bowel sounds (2), and serosanguineous drainage (4) are acceptable assessment findings for this patient after this procedure and require continued monitoring but do not necessarily require reporting to the surgeon.

A nurse instructs a patient to breathe deeply to open collapsed alveoli. Which statement should the nurse offer the patient to best explain how this procedure improves oxygenation? 1. "The alveoli need oxygen to live." 2. "The alveoli have no direct effect on oxygenation." 3. "Collapsed alveoli increase oxygen demands." 4. "Oxygen is exchanged for carbon dioxide in the alveolar membrane."

4. "Oxygen is exchanged for carbon dioxide in the alveolar membrane." Rationale: The exchange of oxygen and carbon dioxide occurs in the alveolar membrane. Therefore, if the alveoli collapse, this exchange cannot occur. Explaining this process in simple terms to a patient may increase compliance with recommended breathing exercises aimed at improving oxygenation. Alveoli do have a direct effect on oxygenation. Answer options 1 and 3 are nonspecific regarding the pathophysiology of the alveolar membrane.

A camp counselor takes an 8-year-old child to see the nurse after playing near what is believed to be poison ivy. On assessment, the child has vesicles on the arms and legs. Which of the following best describes the findings? 1. A lesion filled with purulent drainage. 2. An erosion into the dermis. 3. A solid mass of fibrous tissue. 4. A lesion filled with serous fluid.

4. A lesion filled with serous fluid. Rationale: A vesicle is a small blisterlike elevation on the skin containing serous fluid. Vesicles are usually transparent. Common causes of vesicles include herpes, herpes zoster, and dermatitis associated with poison oak or ivy. A lesion filled with purulent drainage is known as a pustule; an erosion into the dermis is known as an excoriation or ulcer; and a solid mass of fibrous tissue is known as a papule.

A nurse assesses a patient with dry, brittle hair; flaky skin; a beefy-red tongue; and bleeding gums. The nurse should recognize that these clinical manifestations are most likely a result of: 1. A food allergy. 2. Noncompliance with medications. 3. Side effects from medications. 4. A nutritional deficiency.

4. A nutritional deficiency Rationale: All the signs listed are classic for a poor nutritional state lacking in basic nutrients such as vitamins and protein. A specific food allergy or medication (1, 2, 3) is not described, therefore there is not enough information to assume the signs and symptoms are related to either.

A patient who had a cardiac catheterization through the femoral artery is found to have a large amount of blood under his buttocks. After donning gloves, which action should the nurse take first? 1. Apply pressure to the site. 2. Obtain vital signs. 3. Change the patient's gown and bed linens. 4. Assess the catheterization site.

4. Assess the catheterization site. Rationale: Observing standard precautions is the first priority when dealing with any body fluid, followed by assessment of the catheterization site as the second priority. This action establishes the source of the blood and determines how much blood has been lost. Once the source of the bleeding is determined the priority goal for this patient is to stop the bleeding and ensure stability of the patient by monitoring the vital signs. Changing the patient's gown and bed linens is not necessary until the bleeding is controlled and the patient is stabilized.

A patient refuses to follow the physician's orders and leaves the hospital against medical advice. What risk is the patient assuming? 1. Acting irresponsibly 2. Violating the physician's orders 3. Contributing to negligence 4. Assuming the risk for his health state

4. Assuming the risk for his health state Rationale: The patient has the right to self-determination, which includes refusing medical treatment. However, if he does, he must accept full responsibility for his illness and possible injury or undesirable outcomes. Health care professionals have a responsibility to inform the patient and, if possible, have him sign an informed waiver or a leaving against medical advice document. Acting irresponsibly and contributing to negligence are subjective assumptions. The patient may be violating doctor's orders, however if the patient is deemed competent, they have the right to refuse treatment.

The physician orders tap water enemas until clear. The nurse should give no more than 2 enemas to prevent the occurrence of which condition? 1. Hypercalcemia 2. Hypocalcemia 3. Hyperkalemia 4. Hypokalemia

4. Hypokalemia Rationale: Repeated tap water enemas deplete cells and extracellular fluid of potassium and sodium resulting in hypokalemia, hyponatremia, and the potential for water intoxication. Repeated tap water enemas do not have a direct effect on hyper- or hypocalcemia. Potassium is depleted from cells and extracellular fluid and does not result in hyperkalemia.

Which factor has the greatest influence on diastolic blood pressure? 1. Renal function. 2. Cardiac output. 3. Oxygen saturation. 4. Peripheral vascular resistance

4. Peripheral vascular resistance Rationale: Peripheral vascular resistance is the impedance of blood flow, or back pressure, by the arterioles, which is the most influential component of diastolic blood pressure. Renal function through the renin-angiotensin-aldosterone system regulates fluid balance and does influence blood pressure. Cardiac output is the determinant of systolic blood pressure. Oxygen saturation does not have a direct effect on diastolic blood pressure.

When providing preoperative teaching for a patient having surgery, the nurse should focus primarily on which area? 1. Helping the patient decide if surgery is necessary. 2. Providing emotional support to the patient and family. 3. Giving minute-by-minute details of the surgery to the patient and family. 4. Providing general information to reduce patient and family anxiety.

4. Providing general information to reduce patient and family anxiety. Rationale: The primary role of the nurse during preoperative teaching is to provide general information about the surgical experience and what to expect before and after surgery. Helping the patient decide if surgery is necessary (1) is not an appropriate intervention for the nurse. It is also not appropriate for the nurse to describe minute-by-minute details of surgery (3) unless the patient and family request this information, at which time the surgeon should answer the questions. Emotional support (2) is important and would be included as part of providing general information to reduce patient and family anxiety.

A patient has a nursing diagnosis of risk for infection. What would be the most desirable expected outcome for this patient? 1. All nursing functions will be completed by discharge. 2. All invasive intravenous lines will remain patent. 3. The patient will remain awake, alert, and oriented at all times. 4. The patient will be free of signs and symptoms of infection by discharge.

4. The patient will be free of signs and symptoms of infection by discharge. Rationale: Whenever a patient has an infection or is at risk for infection, the nurse's primary objective in providing care is to prevent infection or perform activities that will promote the patient's being free from infection by the time of discharge. The other expected outcomes (1,2,3) are desirable but are more general in nature.

The circulating nurse in the operating room positions a patient for surgery. Which of the following is incorrect regarding proper positioning during surgery? 1. Proper positioning provides optimum exposure to the surgical site. 2. Proper positioning intraoperatively maintains body alignment and protect bony prominences. 3. The patient's position during surgery allows for airway maintenance and access for medication administration. 4. The patient's position for the surgical procedure is determined solely by the circulating nurse.

4. The patient's position for the surgical procedure is determined solely by the circulating nurse. Rationale: The surgeon's preference for the patient's intraoperative position is taken into consideration. Therefore, it is key that the circulating nurse communicate with the surgeon regarding the surgical approach required and their preference. Correct positioning during surgery ensures optimum exposure to the surgical site (1) as well as maintenance of proper body alignment and protection of bony prominences (2). Proper positioning also supports airway maintenance and access for medications (3).

A patient passes black, tarry stools. The nurse recognizes that this may be an indication of: 1. Hemorrhoids, internal and external. 2. An overproduction of bile. 3. Lower gastrointestinal bleeding. 4. Upper gastrointestinal bleeding.

4. Upper gastrointestinal bleeding Rationale: Black, tarry stool (also known as melena) is indicative of an upper GI bleed. The black, tarry appearance and consistency are related to digestive effects on blood caused by breakdown of red blood cells (RBCs). Upper GI bleeding also presents with a distinctive foul smell that is not found with lower GI bleeding. Hemorrhoids and lower GI bleeding are characterized by red blood in the stool. An overproduction of bile is not related to black, tarry stool presentation.

A patient is ordered to receive an IV infusion of potassium chloride (KCl) 40 mEq in 100 mL of 5% dextrose and water to be infused over 2 hours. Before administering this IV medication it is a priority for the nurse to assess which of the following? Select all that apply. 1. Urinary output 2. Deep tendon reflexes 3. Last bowel movement 4. Arterial blood gas results 5. Last serum potassium level 6. Patency of the intravenous access

Correct answers: 1, 5, 6 Rationale: Before administering IV potassium, the urinary output must be normal. If the urine output is low, a potential infusion may damage renal cells. The last serum potassium level should also be checked to ensure potassium replacement is appropriate. A patent IV access is essential, because potassium is very irritating and painful to subcutaneous tissue. The infusion of KCL 40mEq in 100 mL of 5% dextrose and water has no direct effect on deep tendon reflexes, bowel movement patterns, or arterial blood gases. Therefore, these items are not required to be assessed prior to administration of this medication.

Glasgow Coma Scale

Eye opening response: 4=spontaneous 3=to verbal stimuli 2=to pain 1=none Verbal response: 5=orientated 4=confused 3=inappropriate words 2=incoherent 1=none Motor response: 6=obeys commands 5=localizes to pain 4=withdraws from pain 3=flexion to pain (decorticate) 2=extension to pain (decerebrate) 1=none


Conjuntos de estudio relacionados

MIDTERM General Clinical Ortho, Ankle and Foot, Joint Mob, Soft Tissue Mobilization, Documentation, Clinical Ortho Knee, Hip, Peds, Spine

View Set

Chapter 14 - Developing Price Strategies and Programs

View Set

Financial Accounting Practice Exam #1

View Set

English Research Paper Notecards

View Set